Converting a Cartesian Integral to a Polar Integral

Click For Summary
The discussion focuses on converting a Cartesian integral to polar coordinates for the area defined by the semicircle x = √4 - y^2 and the line x = y. The intersection point of these graphs is (√2, √2), with y ranging from √2 to 2. The correct polar integral is presented as ∫∫ r dr dΘ, with r limits from sec(Θ) to 2 sin(Θ) and Θ from π/4 to π/2. The confusion arises from determining the correct lower limit for r, which is clarified through the relationship between r, y, and the angle Θ. This conversion highlights the importance of accurately identifying the limits of integration in polar coordinates.
Amadeo
Messages
28
Reaction score
9
Homework Statement
Convert the following double integral to polar coordinated and evaluate:
Relevant Equations
x=rcosΘ
y=rsinΘ
Q16.PNG


the graph of x= √4-y^2 is a semicircle or radius 2 encompassing the right half of the xy plane (containing points (0,2); (2,0); (0-2))
the graph of x=y is a straight line of slope 1

The intersection of these two graphs is (√2,√2)

y ranges from √2 to 2. Therefore, the area over which we integrate is between the line y=√2; y=2; and x=√4-y^2 as in the shaded region in this picture:

IMG_1154.JPG


This means that the new integral, in polar form is:

∫∫ r dr dΘ with r ranging from √2cscΘ to 2 and Θ ranging from π/4 to π/2
However, the solution is

∫∫ r dr dΘ with r ranging from 2cscΘ to 2 and Θ ranging from π/4 to π/2

I do not understand why this is the case. If y=√2, then rsinΘ = √2 => r=(√2)cscΘ. this would then be the lower limit, as r extends out to the upper limit of 2. Thank you for your assistance.
 
Physics news on Phys.org
From your drawing, it looks to me as though you are letting ##x## vary between ##0## and ##\sqrt {4-y^2}##.
 
:doh: Oh gosh, how embarrassing. Thank you.
 
The first thing I notice is that (\sqrt{2},\sqrt{2}) lies on the line \theta= \frac{\pi}{4} and that (0, 2) lies on the line \theta= \frac{\pi}{2}. So the "\theta" integral is from \frac{\pi}{4} to \frac{\pi}{2}. Now, for each \theta, r is measured on the line through the origin making angle \theta with the x-axis so with slope tan(\theta): y= tan(\theta)x. That line crosses y= \sqrt{2} where x= \frac{\sqrt{2}}{tan(\theta)}. r is the distance from the origin to that point: r= \sqrt{2+ 2 tan^2(\theta)}= \sqrt{2}\sqrt{1+ tan^2(\theta)}= sec(\theta). That line crosses the circle x^2+ y^2= 4 where x^2+ tan^2(\theta)x^2= sec^2(\theta)x^2= 4 or x= 2 cos(\theta), y= 2 cos(\theta)tan(\theta)= 2 sin(\theta). <br /> <br /> The integral is \int_{\pi/4}^{\pi/2} \int_{sec(\theta)}^{2 sin(\theta)} r dr d\theta.
 
Question: A clock's minute hand has length 4 and its hour hand has length 3. What is the distance between the tips at the moment when it is increasing most rapidly?(Putnam Exam Question) Answer: Making assumption that both the hands moves at constant angular velocities, the answer is ## \sqrt{7} .## But don't you think this assumption is somewhat doubtful and wrong?

Similar threads

  • · Replies 19 ·
Replies
19
Views
2K
  • · Replies 3 ·
Replies
3
Views
2K
  • · Replies 3 ·
Replies
3
Views
1K
  • · Replies 21 ·
Replies
21
Views
3K
Replies
3
Views
1K
  • · Replies 2 ·
Replies
2
Views
2K
  • · Replies 1 ·
Replies
1
Views
2K
  • · Replies 2 ·
Replies
2
Views
2K
  • · Replies 1 ·
Replies
1
Views
1K
  • · Replies 12 ·
Replies
12
Views
2K